Difference between revisions of "1962 AHSME Problems/Problem 40"
(→Solution) |
(→Solution) |
||
Line 5: | Line 5: | ||
==Solution== | ==Solution== | ||
− | {{ | + | The series can be written as the following: |
+ | |||
+ | <math>\frac{1}{10} + \frac{1}{10^2} + \frac{1}{10^3} + ...</math> | ||
+ | |||
+ | <math>+ \frac{1}{10^2} + \frac{1}{10^3} + \frac{1}{10^4} + ...</math> | ||
+ | |||
+ | <math>+ \frac{1}{10^3} + \frac{1}{10^4} + \frac{1}{10^5} + ...</math> | ||
+ | |||
+ | and so on. | ||
+ | |||
+ | by using the formula for infinite geometric series <math>(\frac{a}{1-r})</math>, | ||
+ | |||
+ | We can get <math>\frac{\frac{1}{10}}{1-\frac{1}{10}}</math> <math>+</math> <math>\frac{\frac{1}{10^2}}{1-\frac{1}{10}}</math> <math>+</math> <math>\frac{\frac{1}{10^3}}{1-\frac{1}{10}}</math> <math>+</math> ... | ||
+ | Since they all have common denominators, we get <math>\frac{(\frac{1}{10} + \frac{1}{10^2} + \frac{1}{10^3})}{\frac{9}{10}}</math>. | ||
+ | Using the infinite series formula again, we get <math>\frac{\frac{\frac{1}{10}}{1-\frac{1}{10}}}{\frac{9}{10}}</math> <math>=</math> <math>\frac{\frac{\frac{1}{10}}{\frac{9}{10}}}{\frac{9}{10}}</math> <math>=</math> <math>\frac{\frac{1}{9}}{\frac{9}{10}}</math> <math>=</math> <math>\boxed{ (B) \frac{10}{81}}</math> |
Revision as of 19:27, 19 February 2016
Problem
The limiting sum of the infinite series, whose th term is is:
Solution
The series can be written as the following:
and so on.
by using the formula for infinite geometric series ,
We can get ... Since they all have common denominators, we get . Using the infinite series formula again, we get